Find the Median of the set of integers A? 1) Mean of the se

This topic has expert replies
User avatar
Legendary Member
Posts: 1100
Joined: Sat May 10, 2014 11:34 pm
Location: New Delhi, India
Thanked: 205 times
Followed by:24 members
Find the Median of the set of integers A?

1) Mean of the set A is 10
2) Range of the set A is 2

Source: www.GMATinsight.com

Answer: option C
"GMATinsight"Bhoopendra Singh & Sushma Jha
Most Comprehensive and Affordable Video Course 2000+ CONCEPT Videos and Video Solutions
Whatsapp/Mobile: +91-9999687183 l [email protected]
Contact for One-on-One FREE ONLINE DEMO Class Call/e-mail
Most Efficient and affordable One-On-One Private tutoring fee - US$40-50 per hour

User avatar
Legendary Member
Posts: 2663
Joined: Wed Jan 14, 2015 8:25 am
Location: Boston, MA
Thanked: 1153 times
Followed by:128 members
GMAT Score:770

by DavidG@VeritasPrep » Mon May 22, 2017 8:08 am
GMATinsight wrote:Find the Median of the set of integers A?

1) Mean of the set A is 10
2) Range of the set A is 2

Source: www.GMATinsight.com

Answer: option C
1) Set 1: {10,10} Median = 10
Set 2: {0,0,30} Median = 0
Not sufficient

2)Set 1: {1,3} Median = 2
Set 2: {2, 4} Median = 3
Not sufficient

Together: Notice how important it is that we're dealing with integers - If the range is 2, this dictates that our high and low will have to be 9 and 11, otherwise the mean won't be 10.

Set 1: {9,11} Median = 10
Set 2: {9, 10, 11} Median = 10
Set 3: {9, 9, 9, 11, 11, 11} Median = 10

No matter set we assemble, the median will always be 10. Sufficient. The answer is C
Veritas Prep | GMAT Instructor

Veritas Prep Reviews
Save $100 off any live Veritas Prep GMAT Course

GMAT/MBA Expert

User avatar
GMAT Instructor
Posts: 3008
Joined: Mon Aug 22, 2016 6:19 am
Location: Grand Central / New York
Thanked: 470 times
Followed by:34 members

by Jay@ManhattanReview » Mon May 29, 2017 10:24 pm
GMATinsight wrote:Find the Median of the set of integers A?

1) Mean of the set A is 10
2) Range of the set A is 2

Source: www.GMATinsight.com

Answer: option C
Statement 1: Mean of Set A is 10.

Clearly insufficient.

Say, for example, Set A: {10, 10, 10}; Mean = 10 and Median = 10; however, if Set A: {0, 0, 30}; Mean = 10 and Median = 0. No unique value. Insufficient.

Statement 2: Range of Set A is 2.

Clearly insufficient.

Say, for example, Set A: {10, 10, 12}; Median = 10; however, if Set A: {20, 20, 22}; Median = 20. No unique value. Insufficient.

Statement 1 and 2:

Since the mean of the set is 10, the maximum value of an element of the set must be at least 10, thus the minimum value of an element of the set must be at least 10 - 2 = 8.

We cannot have a set such that the minimum value of an element equals 8, and have mean = 10. Thus, the minimum value of an element must be 9; this follows that the maximum value of an element of the set must be 11.

If there are Odd numbers of elements in the set, the middlemost element must 10, or Median = 10; however, if there are Even numbers of elements in the set, there is no middlemost element, thus Median = Average of 9 and 11 = 10. In each case Median = 10. Sufficient.

The correct answer: C

-Jay
_________________
Manhattan Review GMAT Prep

Locations: New York | Singapore | Doha | Lausanne | and many more...

Schedule your free consultation with an experienced GMAT Prep Advisor! Click here.